steven.kantowitz
Thanks Received: 0
Vinny Gambini
Vinny Gambini
 
Posts: 10
Joined: March 24th, 2010
 
 
 

Q21 - The new agrilcultural bill will

by steven.kantowitz Wed May 05, 2010 1:24 pm

I understand why A is correct, but why is E incorrecy?
User avatar
 
ManhattanPrepLSAT1
Thanks Received: 1909
Atticus Finch
Atticus Finch
 
Posts: 2851
Joined: October 07th, 2009
 
This post thanked 4 times.
 
 

Re: Q21 - The new agrilcultural bill will

by ManhattanPrepLSAT1 Thu May 06, 2010 12:28 am

What's the difference in knowing the following two pieces of information?

1. The percentage of bills not supported that ended up passing.
2. The percentage of bills passed that had been supported.

The evidence is: the bill is not supported by any major party.

The conclusion is: the bill will fail to pass.

The gap in the reasoning is: if a bill is not supported then it will not pass.

None of the answer choices gives us a conditional relationship that bridges the gap. However we do get a likelihood. Each of the answer choices says "MOST" except for answer choice (C). Answer choice (C) undermines the argument so we can eliminate this one right away. Bridging the gap with a "MOST" statement is answer choice (A). Answer choice (E) would be similar to a contrapositive of a "MOST" statement - which is not permitted.

The first statement above is what (A) gives us, whereas the second is what (E) gives us. We need the information in (A) to bridge the gap.

If you happen to see this more formally...


~Supported
------------------
~Pass

Gap: ~Supported --> ~Pass


(A) says: ~Supported most ~Pass
(E) says: Pass most Supported

Answer choice (E) is similar to a contrapositive of a most statement and (B) is the reversal of a most statement. Both would be typical answer choices set up to make you ask, "How is the relationship supposed to be stated exactly?"
 
ebrickm2
Thanks Received: 2
Forum Guests
 
Posts: 44
Joined: March 07th, 2010
 
 
 

Re: PT 30, S2, Q21 - The new agrilcultural bill will

by ebrickm2 Sat Jul 24, 2010 9:06 pm

Is it only b/c you can't do a contrapositive of a most statement that makes E wrong, otherwise I still don't see it based on your explanation.
User avatar
 
ManhattanPrepLSAT1
Thanks Received: 1909
Atticus Finch
Atticus Finch
 
Posts: 2851
Joined: October 07th, 2009
 
This post thanked 3 times.
 
 

Re: PT 30, S2, Q21 - The new agrilcultural bill will

by ManhattanPrepLSAT1 Sun Jul 25, 2010 11:19 am

That is exactly why answer choice (E) can be eliminated. You cannot take the contrapositive of a "some" statement nor a "most" statement.

Frustrating I know, because so many of the answer choices are very close, but it's really important that we do not commit reversals, negations, or incorrectly taken contrapositives!

Does that clear things up on this one?
 
ebrickm2
Thanks Received: 2
Forum Guests
 
Posts: 44
Joined: March 07th, 2010
 
 
 

Re: PT 30, S2, Q21 - The new agrilcultural bill will

by ebrickm2 Sun Jul 25, 2010 8:34 pm

What a subtle jerk of a question, I love it.

Thanks for the explanation.
 
bigtree65
Thanks Received: 2
Forum Guests
 
Posts: 38
Joined: September 16th, 2011
 
 
 

Re: Q21 - The new agrilcultural bill will

by bigtree65 Sat Oct 01, 2011 10:32 pm

Yea I ruled out A and E immediately because I thought they were contrapositives and fell for B, thanks for the explanation I won't make that mistake again.
 
mcrittell
Thanks Received: 5
Atticus Finch
Atticus Finch
 
Posts: 154
Joined: May 25th, 2011
 
 
 

Re: Q21 - The new agrilcultural bill will

by mcrittell Tue Oct 18, 2011 11:12 am

What would a contrapositive of a "most" statement look like? I've never heard of this. Thanks
User avatar
 
ManhattanPrepLSAT1
Thanks Received: 1909
Atticus Finch
Atticus Finch
 
Posts: 2851
Joined: October 07th, 2009
 
 
 

Re: Q21 - The new agrilcultural bill will

by ManhattanPrepLSAT1 Tue Oct 18, 2011 5:24 pm

I think that's the point I was trying to make. There is no such thing as a contrapositive of a "most statement."

I notate "most A's are B's":

A most> B

I also use "some statements" and would notate "some A's are B's":

A some B

There are no contrapositives of either, but sometimes I've seen questions where the error in the reasoning is the attempt of a contrapositive.

A most> B

interpreted to be

~B most> ~A

Does that answer your question?
 
nmop_apisdn2
Thanks Received: 16
Vinny Gambini
Vinny Gambini
 
Posts: 24
Joined: June 23rd, 2012
 
This post thanked 2 times.
 
 

Re: Q21 - The new agrilcultural bill will

by nmop_apisdn2 Tue Jul 24, 2012 12:46 am

Ran here to post my thoughts on the question:

I really just want to jump straight into the argument.

(Conclusion)The new bill will almost surely fail to pass.
(Premise)The leaders of all major parties have stated that they oppose it.

PIL = Passed into Law
S = Supported it.
and the "-" denotes a negation.

Diagrammed:

-PIL (conclusion)
-S (premise)

Rearranged, we get:

-S (They dont support it, since they all oppose it.)
--------------------------------------------------------
Therefore,
- PIL (Therefore, it won't pass into law.)

The answer choices:

(A) is correct because when diagrammed, you get:

Most -S --> -PIL (most not supported bills are not passed into law)
What this answer choice is essentially doing is establishing the conditional that, when the antecedent in this answer choice has been confirmed per the premise in the stimulus, it must be the case that the consequent follows.

I'll plug it into the argument to show you what I'm talking about.

The conditional claim being made in this answer choice:
most -S --> - PIL

The stimulus:

-S (The stimulus' only premise)

most -S --> -PIL (the conditional claim in the answer choice)

Therefore, -PIL.

This type of logic is valid and is known as affirming the antecedent, which always leads to affirming the consequent.

(B) is incorrect because it is the opposite of the logic we are looking for. This answer choice, when diagrammed, is:

most -PIL --> -S

You would basically be affirming the consequent in order to lead to the conclusion, which is a type of invalid logic. The Manhattan book says that the shift in the term "leaders" to "members" is what makes this wrong, but I find that there is a more logical basis than this for eliminating this answer choice. It is an invalid form of logic to try and affirm the consequent to reach the affirmed antecedent.

(C) is incorrect because we are talking about how the leaders will not pass the bill into law, and not talking about how to pass a bill into law.
(D) is incorrect because again, we are not talking about how past bills have been passed into law; we are looking for something that bridges the gap in the argument about NOT passing the bill into law. Basically the same reason for eliminating (C).
(E) is incorrect for the same reasons as (C) and (D). Always remember to pay attention to what the issue is in the stimulus, and to not be concerned with the opposite issue, because most of the time, the opposite issue is not of our concern and won't help us to reach the most logically strengthening answer choice.

Phew, I guess that Philosophy major did help. lol
 
shodges
Thanks Received: 0
Jackie Chiles
Jackie Chiles
 
Posts: 41
Joined: August 23rd, 2011
 
 
 

Re: Q21 - The new agrilcultural bill will

by shodges Tue May 21, 2013 5:45 pm

nmop_apisdn2 Wrote:Ran here to post my thoughts on the question:

I really just want to jump straight into the argument.

(Conclusion)The new bill will almost surely fail to pass.
(Premise)The leaders of all major parties have stated that they oppose it.

PIL = Passed into Law
S = Supported it.
and the "-" denotes a negation.

Diagrammed:

-PIL (conclusion)
-S (premise)

Rearranged, we get:

-S (They dont support it, since they all oppose it.)
--------------------------------------------------------
Therefore,
- PIL (Therefore, it won't pass into law.)

The answer choices:

(A) is correct because when diagrammed, you get:

Most -S --> -PIL (most not supported bills are not passed into law)
What this answer choice is essentially doing is establishing the conditional that, when the antecedent in this answer choice has been confirmed per the premise in the stimulus, it must be the case that the consequent follows.

I'll plug it into the argument to show you what I'm talking about.

The conditional claim being made in this answer choice:
most -S --> - PIL

The stimulus:

-S (The stimulus' only premise)

most -S --> -PIL (the conditional claim in the answer choice)

Therefore, -PIL.

This type of logic is valid and is known as affirming the antecedent, which always leads to affirming the consequent.

(B) is incorrect because it is the opposite of the logic we are looking for. This answer choice, when diagrammed, is:

most -PIL --> -S

You would basically be affirming the consequent in order to lead to the conclusion, which is a type of invalid logic. The Manhattan book says that the shift in the term "leaders" to "members" is what makes this wrong, but I find that there is a more logical basis than this for eliminating this answer choice. It is an invalid form of logic to try and affirm the consequent to reach the affirmed antecedent.

(C) is incorrect because we are talking about how the leaders will not pass the bill into law, and not talking about how to pass a bill into law.
(D) is incorrect because again, we are not talking about how past bills have been passed into law; we are looking for something that bridges the gap in the argument about NOT passing the bill into law. Basically the same reason for eliminating (C).
(E) is incorrect for the same reasons as (C) and (D). Always remember to pay attention to what the issue is in the stimulus, and to not be concerned with the opposite issue, because most of the time, the opposite issue is not of our concern and won't help us to reach the most logically strengthening answer choice.

Phew, I guess that Philosophy major did help. lol


Awesome explanation. Thanks!
 
dukeag
Thanks Received: 2
Vinny Gambini
Vinny Gambini
 
Posts: 17
Joined: April 22nd, 2014
 
 
 

Re: PT 30, S2, Q21 - The new agrilcultural bill will

by dukeag Thu May 22, 2014 9:30 am

mattsherman Wrote:That is exactly why answer choice (E) can be eliminated. You cannot take the contrapositive of a "some" statement nor a "most" statement.

Frustrating I know, because so many of the answer choices are very close, but it's really important that we do not commit reversals, negations, or incorrectly taken contrapositives!

Does that clear things up on this one?


Why are we not allowed to take the contrapositive of "some" or "most" statements?
User avatar
 
WaltGrace1983
Thanks Received: 207
Atticus Finch
Atticus Finch
 
Posts: 837
Joined: March 30th, 2013
 
 
trophy
Most Thanked
trophy
Most Thankful
trophy
First Responder
 

Re: Q21 - The new agrilcultural bill will

by WaltGrace1983 Tue Jun 24, 2014 3:54 pm

mattsherman Wrote:None of the answer choices gives us a conditional relationship that bridges the gap. However we do get a likelihood. Each of the answer choices says "MOST" except for answer choice (C). Answer choice (C) undermines the argument so we can eliminate this one right away. Bridging the gap with a "MOST" statement is answer choice (A). Answer choice (E) would be similar to a contrapositive of a "MOST" statement - which is not permitted.


I totally understand why (A) is the BEST answer, which is why I picked it. However, wouldn't (C) be classified as the all-too-famous (absence of a cause) → (absence of an effect) strengthener?

If we have (oppose) → ~(pass) as the argument and (C) gives us ~(oppose) → (pass), wouldn't this actually strengthen the argument in a very LSAT-like way?
 
cwolfington
Thanks Received: 4
Jackie Chiles
Jackie Chiles
 
Posts: 29
Joined: May 15th, 2014
 
 
 

Re: Q21 - The new agrilcultural bill will

by cwolfington Thu Jun 26, 2014 1:15 am

I found it helpful to prephrase an answer for this question. I originally diagrammed the stimulus with formal logic, but after reading the answers I decided to look at the authors assumption, which is, "If no major party leader supports a bill, it will almost surely fail to pass" Answer choice (A), if true, strengthens the author's assumption.
 
christine.defenbaugh
Thanks Received: 585
Atticus Finch
Atticus Finch
 
Posts: 536
Joined: May 17th, 2013
 
This post thanked 1 time.
 
 

Re: PT 30, S2, Q21 - The new agrilcultural bill will

by christine.defenbaugh Tue Jul 22, 2014 1:41 pm

dukeag Wrote:
mattsherman Wrote:That is exactly why answer choice (E) can be eliminated. You cannot take the contrapositive of a "some" statement nor a "most" statement.

Frustrating I know, because so many of the answer choices are very close, but it's really important that we do not commit reversals, negations, or incorrectly taken contrapositives!

Does that clear things up on this one?


Why are we not allowed to take the contrapositive of "some" or "most" statements?



Great question, dukeag!

The only reason we are allowed to make contrapositives of conditional statements is because they logically equivalent statements. They mean the EXACT same thing! When I state a rule:
If I eat pizza, then I get sick.

That also means that if I DON't get sick, then I must NOT have eaten pizza....because if I *had* eaten pizza, I would have gotten sick!

The rule and its contrapositive are logically identical.

If you tried to take the contrapositive of a most statement, the odd statement you'd end up creating would NOT be logically identical.

Consider the (factually correct) statement: Most drivers are women.

If we attempted to created a contrapositive of that, it might look something like this: Most men are not drivers.

And that's ridiculous!

Or consider a class where there are 20 students, 10 boys and 10 girls. 8 of the boys got As, while 9 of the girls got As. It would be correct to say that most of the As were girls, but what might seem to be the 'contrapositive' ("most of the boys got non-As") would clearly be wrong.

On a fundamental level, taking the contrapositive of a conditional only works because that conditional is a guarantee of some outcome. As a result, taking that outcome away will also tell us something, since it was promised in certain situations. Statements about 'most', on the other hand, don't guarantee anything at all - they just give us a squishy likelihood. Taking away the 'likely' result, then, doesn't actually tell us anything.

WaltGrace1983 Wrote:
mattsherman Wrote:None of the answer choices gives us a conditional relationship that bridges the gap. However we do get a likelihood. Each of the answer choices says "MOST" except for answer choice (C). Answer choice (C) undermines the argument so we can eliminate this one right away. Bridging the gap with a "MOST" statement is answer choice (A). Answer choice (E) would be similar to a contrapositive of a "MOST" statement - which is not permitted.


I totally understand why (A) is the BEST answer, which is why I picked it. However, wouldn't (C) be classified as the all-too-famous (absence of a cause) → (absence of an effect) strengthener?

If we have (oppose) → ~(pass) as the argument and (C) gives us ~(oppose) → (pass), wouldn't this actually strengthen the argument in a very LSAT-like way?


Bah! I've always thought those tidy shortcuts of 'absence of cause, absence of effect' strengtheners were dangerous thought patterns, and this is exactly why.

Let me lay out a few examples that should illustrate the point.

    PREMISE: Bees are dying in California
    CONCLUSION: It must be because of pollution.


Here, it would totally strengthen this argument to say "In Utopia, there's zero pollution and the bees aren't dying." Absence of proposed cause, absence of observed effect.

    PREMISE: Bees are dying in California. California has lots of pollution
    CONCLUSION: It must be because of pollution.


Here, again, it would totally strengthen this argument to say "In Utopia, there's zero pollution and the bees aren't dying." Absence of observed cause, absence of observed effect.

    PREMISE: There's lots of pollution in California
    CONCLUSION: Bees must be dying there.


Now, it has absolutely zero impact on the argument to say that "In Utopia, there's zero pollution and the bees aren't dying." Absence of observed cause, absence of proposed effect means nothing. Why? Because, while this argument is about cause/effect, saying that we KNOW we have the cause element, and we are PREDICTING some effect in the future. It's not suggesting it's the only possible cause for that effect, so removing the cause in Utopia doesn't mean much to us.

The first two examples started from a known effect, and sought an explanation. The only way to conclude definitively that the effect *was* caused by a particular cause is to assume that no other cause could have done it. As a result, it's useful information to see what happens when we take the supposedly sole cause away.

If we want to be formal about it, when we know the cause only, the conditional relationship is:
    If [cause] --> [effect]

Just like a bad contrapositive, finding out what happens when we negate the cause isn't useful.

However, when we know the effect has happened, and we look for a cause, the conditional relationship is:
    If [effect] --> [cause]

And the contrapositive of that is: If NO [cause] --> NO [effect] And that's exactly where the 'absence of cause, absence of effect' shortcut came from.

Moral of the story Causal relationships must be handled with care, and they function a bit differently depending on whether we know the cause or the effect actually occurred.
 
mharr
Thanks Received: 1
Forum Guests
 
Posts: 28
Joined: January 07th, 2014
 
 
 

Re: Q21 - The new agrilcultural bill will

by mharr Mon Feb 23, 2015 9:05 pm

I apologize for the silly question. Would someone mind explaining how answer choice C undermines the argument? The answer choice seems irrelevant to me because it addresses support for a bill and how it passes because of the support. The stimulus, however, addresses no support for a bill and how it will not pass.
 
daijob
Thanks Received: 0
Elle Woods
Elle Woods
 
Posts: 74
Joined: June 02nd, 2015
 
 
 

Re: Q21 - The new agrilcultural bill will

by daijob Thu Jul 09, 2015 10:24 pm

I chose B because I thought this bridges the gap and thought this is the contrapositive of A. I did not choose A because I thought contrapositive statement for MOST statement can involve "at least" so, I chose B which includes it...But there is no contrapositive for MOST statement and it does not matter whether it includes "at least" or not...right??

Btw,
I'm also confused with the CAUSE/EFFECT statement...
So If CAUSE=> then EFFECT is true,
NO EFFECT=> NO CAUSE, and did you say if EFFECT=> then CAUSE is also true??
Or did I mis-read something?
User avatar
 
rinagoldfield
Thanks Received: 309
Atticus Finch
Atticus Finch
 
Posts: 390
Joined: December 13th, 2011
 
 
 

Re: Q21 - The new agrilcultural bill will

by rinagoldfield Wed Jul 15, 2015 5:19 pm

Thanks for your post, Daijob. (B) is incorrect because it goes backwards from the conclusion to the premise, while (A) correctly flows from the premise to the conclusion. In addition, as Matt stated above, there is no such thing as a “most” contrapositive.

Don’t worry too much about the cause-effect stuff. That’s really far into the weeds and irrelevant to this question. The key takeaway for this question is that you can’t take a contrapositive of a “most” statement!
 
roflcoptersoisoi
Thanks Received: 0
Atticus Finch
Atticus Finch
 
Posts: 165
Joined: April 30th, 2015
 
 
 

Re: Q21 - The new agrilcultural bill will

by roflcoptersoisoi Fri Aug 05, 2016 8:23 pm

LMP --> ~S
------------------
NAB --> ~P

Assumption:: (LMP --> ~S ) ---> (NAB ---> ~S)

To get from NAB to ~P, we must link NAB to LMP and ~S to ~P, so we get: NAB --> LMP --> ~S ---> ~P

The missing link will at least partially bridge the gap between: NAB and P or ~S and ~P

(A) ~S --(most)--> ~P, bingo.
(B) ~P --(most)--> ~S logical reversal, invalid inference
(C) LMP --> S -> P, another logical reversal and invalid inference
(D) P ----(most)---> ~S logical reversal, invalid invalid inference
(E) P --(most)--> S , logical reversal, invalid inference
 
ChentuoZ870
Thanks Received: 0
Vinny Gambini
Vinny Gambini
 
Posts: 10
Joined: January 25th, 2022
 
 
 

Re: Q21 - The new agrilcultural bill will

by ChentuoZ870 Mon Mar 28, 2022 5:21 am

Some more discussion about why contrapositive does not apply to most statements.

Consier the following example:

9 bills, supported by at least one leader of a major party, passed.
40 bills, supported by at least one leader of a major party, not passed.
1 bills, not supported by any one leader of a major party, passed.
0 bills, not supported by any one leader of a major party, not passed.

So, together, there are 10 bills passed, and the majority of the bill ( 9/10 ) supported by at least on leader of a major party.
Yet for bills not supported by any one leader of a major party, they all passed(1/1). And that is contrary to the stimulus.

Examples above is consistent with (E), and it demonstrate why E cannot provide the most support for the prediction.